click below
click below
Normal Size Small Size show me how
Question Types
Question | Answer |
---|---|
Which one of the following can be properly inferred from the passage? | MUST BE TRUE |
If the statements above are true, which one of the following must also be true? | MUST BE TRUE |
The main point of the argument is that... | MAIN POINT |
Larew and Mendota disagree about whether | POINT AT ISSUE |
Which one of the following is an assumption required by the argument above? | ASSUMPTION |
Which one of the following, if assumed, allows the conclusion above to be properly drawn? | JUSTIFY THE CONCLUSION |
Which one of the following, if true, most strengthens the argument? | STRENGTHEN |
Which one of the following, if true, most strongly supports the statement above? | STRENGTHEN |
Which one of the following, if true, would most effectively resolve the apparent paradox above? | PARADOX |
Which one of the following, if true, most seriously weakens the argument? | WEAKEN |
Which one of the following describes the technique of reasoning used above? | METHOD |
The reasoning in the astronomer’s argument is flawed because this argument | FLAW |
Which one of the following arguments is most similar in its pattern of reasoning to the argument above? | PARALLEL |
The answer to which one of the following questions would contribute most to an evaluation of the argument?” | EVALUATE THE ARGUMENT |
If the statements above are true, which one of the following CANNOT be true? | CANNOT BE TRUE |
Which one of the following, if true, most helps to explain the viewpoint of the historians described above? | RESOLVE |
Which one of the following can be properly inferred from Rosen’s statement? | MUST BE TRUE |
Which one of the following, if true, most seriously weakens the reasoning above? | WEAKEN |
Which one of the following is an assumption required by the argument above? | ASSUMPTION |
Which one of the following arguments is most similar in its pattern of reasoning to the argument above? | PARALLEL |
Of the following, which one most accurately expresses the main point of the argument? | MAIN POINT |
Which one of the following, if true, would provide the most support for the economists’ assertion? | STRENGTHEN |
The argument is flawed because it | FLAW |
The dialogue most supports the claim that Tony and Raoul disagree about whether | POINT AT ISSUE |
If the statements above are true, which one of the following must be false? | CANNOT BE TRUE |
The advertisement proceeds by | METHOD |
Which one of the following, if assumed, would allow the conclusion to be properly drawn? | JUSTIFY |
The answer to which one of the following questions would most help in evaluating the columnist’s argument? | EVALUATE |
Sue challenges Anne’s reasoning by | METHOD |
The statements above, if true, most strongly support which one of the following? | MUST BE TRUE |